$underset{xrightarrowinfty}{lim}frac{f(x)}{x}=0$ Implies $underset{xrightarrowinfty}{f'(x)}=0$ [duplicate]












4












$begingroup$



This question already has an answer here:




  • Proving that $limlimits_{xtoinfty}f'(x) = 0$ when $limlimits_{xtoinfty}f(x)$ and $limlimits_{xtoinfty}f'(x)$ exist

    6 answers




Let $f:mathbb{R}rightarrowmathbb{R}$ be a continuously differentiable function such that $underset{xrightarrowinfty}{lim}frac{f(x)}{x}=0$ and suppose $underset{xrightarrowinfty}{f'(x)}$ exists. Then Prove that $underset{xrightarrowinfty}{f'(x)}=0$



I can see that if we apply L'hoptal's theorem directly to $frac{f(x)}{x}$ then we can get the answer. But is it possible to do so without knowing the value of $underset{xrightarrowinfty}{f(x)}$



On the similar problem: found here, they have given the existence of $lim_{xrightarrowinfty} f(x)$. But in this particular problem they haven't










share|cite|improve this question











$endgroup$



marked as duplicate by Math Lover, Cesareo, José Carlos Santos calculus
Users with the  calculus badge can single-handedly close calculus questions as duplicates and reopen them as needed.

StackExchange.ready(function() {
if (StackExchange.options.isMobile) return;

$('.dupe-hammer-message-hover:not(.hover-bound)').each(function() {
var $hover = $(this).addClass('hover-bound'),
$msg = $hover.siblings('.dupe-hammer-message');

$hover.hover(
function() {
$hover.showInfoMessage('', {
messageElement: $msg.clone().show(),
transient: false,
position: { my: 'bottom left', at: 'top center', offsetTop: -7 },
dismissable: false,
relativeToBody: true
});
},
function() {
StackExchange.helpers.removeMessages();
}
);
});
});
Jan 3 at 1:17


This question has been asked before and already has an answer. If those answers do not fully address your question, please ask a new question.















  • $begingroup$
    It is true if you assume that $lim_{x to infty} f'(x)$ exists, see e.g. math.stackexchange.com/questions/42277/…. Also related: math.stackexchange.com/q/313676/42969.
    $endgroup$
    – Martin R
    Jan 2 at 21:35












  • $begingroup$
    Thanks and I edited
    $endgroup$
    – DD90
    Jan 2 at 21:37
















4












$begingroup$



This question already has an answer here:




  • Proving that $limlimits_{xtoinfty}f'(x) = 0$ when $limlimits_{xtoinfty}f(x)$ and $limlimits_{xtoinfty}f'(x)$ exist

    6 answers




Let $f:mathbb{R}rightarrowmathbb{R}$ be a continuously differentiable function such that $underset{xrightarrowinfty}{lim}frac{f(x)}{x}=0$ and suppose $underset{xrightarrowinfty}{f'(x)}$ exists. Then Prove that $underset{xrightarrowinfty}{f'(x)}=0$



I can see that if we apply L'hoptal's theorem directly to $frac{f(x)}{x}$ then we can get the answer. But is it possible to do so without knowing the value of $underset{xrightarrowinfty}{f(x)}$



On the similar problem: found here, they have given the existence of $lim_{xrightarrowinfty} f(x)$. But in this particular problem they haven't










share|cite|improve this question











$endgroup$



marked as duplicate by Math Lover, Cesareo, José Carlos Santos calculus
Users with the  calculus badge can single-handedly close calculus questions as duplicates and reopen them as needed.

StackExchange.ready(function() {
if (StackExchange.options.isMobile) return;

$('.dupe-hammer-message-hover:not(.hover-bound)').each(function() {
var $hover = $(this).addClass('hover-bound'),
$msg = $hover.siblings('.dupe-hammer-message');

$hover.hover(
function() {
$hover.showInfoMessage('', {
messageElement: $msg.clone().show(),
transient: false,
position: { my: 'bottom left', at: 'top center', offsetTop: -7 },
dismissable: false,
relativeToBody: true
});
},
function() {
StackExchange.helpers.removeMessages();
}
);
});
});
Jan 3 at 1:17


This question has been asked before and already has an answer. If those answers do not fully address your question, please ask a new question.















  • $begingroup$
    It is true if you assume that $lim_{x to infty} f'(x)$ exists, see e.g. math.stackexchange.com/questions/42277/…. Also related: math.stackexchange.com/q/313676/42969.
    $endgroup$
    – Martin R
    Jan 2 at 21:35












  • $begingroup$
    Thanks and I edited
    $endgroup$
    – DD90
    Jan 2 at 21:37














4












4








4





$begingroup$



This question already has an answer here:




  • Proving that $limlimits_{xtoinfty}f'(x) = 0$ when $limlimits_{xtoinfty}f(x)$ and $limlimits_{xtoinfty}f'(x)$ exist

    6 answers




Let $f:mathbb{R}rightarrowmathbb{R}$ be a continuously differentiable function such that $underset{xrightarrowinfty}{lim}frac{f(x)}{x}=0$ and suppose $underset{xrightarrowinfty}{f'(x)}$ exists. Then Prove that $underset{xrightarrowinfty}{f'(x)}=0$



I can see that if we apply L'hoptal's theorem directly to $frac{f(x)}{x}$ then we can get the answer. But is it possible to do so without knowing the value of $underset{xrightarrowinfty}{f(x)}$



On the similar problem: found here, they have given the existence of $lim_{xrightarrowinfty} f(x)$. But in this particular problem they haven't










share|cite|improve this question











$endgroup$





This question already has an answer here:




  • Proving that $limlimits_{xtoinfty}f'(x) = 0$ when $limlimits_{xtoinfty}f(x)$ and $limlimits_{xtoinfty}f'(x)$ exist

    6 answers




Let $f:mathbb{R}rightarrowmathbb{R}$ be a continuously differentiable function such that $underset{xrightarrowinfty}{lim}frac{f(x)}{x}=0$ and suppose $underset{xrightarrowinfty}{f'(x)}$ exists. Then Prove that $underset{xrightarrowinfty}{f'(x)}=0$



I can see that if we apply L'hoptal's theorem directly to $frac{f(x)}{x}$ then we can get the answer. But is it possible to do so without knowing the value of $underset{xrightarrowinfty}{f(x)}$



On the similar problem: found here, they have given the existence of $lim_{xrightarrowinfty} f(x)$. But in this particular problem they haven't





This question already has an answer here:




  • Proving that $limlimits_{xtoinfty}f'(x) = 0$ when $limlimits_{xtoinfty}f(x)$ and $limlimits_{xtoinfty}f'(x)$ exist

    6 answers








real-analysis calculus analysis






share|cite|improve this question















share|cite|improve this question













share|cite|improve this question




share|cite|improve this question








edited Jan 2 at 21:52







DD90

















asked Jan 2 at 21:30









DD90DD90

2648




2648




marked as duplicate by Math Lover, Cesareo, José Carlos Santos calculus
Users with the  calculus badge can single-handedly close calculus questions as duplicates and reopen them as needed.

StackExchange.ready(function() {
if (StackExchange.options.isMobile) return;

$('.dupe-hammer-message-hover:not(.hover-bound)').each(function() {
var $hover = $(this).addClass('hover-bound'),
$msg = $hover.siblings('.dupe-hammer-message');

$hover.hover(
function() {
$hover.showInfoMessage('', {
messageElement: $msg.clone().show(),
transient: false,
position: { my: 'bottom left', at: 'top center', offsetTop: -7 },
dismissable: false,
relativeToBody: true
});
},
function() {
StackExchange.helpers.removeMessages();
}
);
});
});
Jan 3 at 1:17


This question has been asked before and already has an answer. If those answers do not fully address your question, please ask a new question.






marked as duplicate by Math Lover, Cesareo, José Carlos Santos calculus
Users with the  calculus badge can single-handedly close calculus questions as duplicates and reopen them as needed.

StackExchange.ready(function() {
if (StackExchange.options.isMobile) return;

$('.dupe-hammer-message-hover:not(.hover-bound)').each(function() {
var $hover = $(this).addClass('hover-bound'),
$msg = $hover.siblings('.dupe-hammer-message');

$hover.hover(
function() {
$hover.showInfoMessage('', {
messageElement: $msg.clone().show(),
transient: false,
position: { my: 'bottom left', at: 'top center', offsetTop: -7 },
dismissable: false,
relativeToBody: true
});
},
function() {
StackExchange.helpers.removeMessages();
}
);
});
});
Jan 3 at 1:17


This question has been asked before and already has an answer. If those answers do not fully address your question, please ask a new question.














  • $begingroup$
    It is true if you assume that $lim_{x to infty} f'(x)$ exists, see e.g. math.stackexchange.com/questions/42277/…. Also related: math.stackexchange.com/q/313676/42969.
    $endgroup$
    – Martin R
    Jan 2 at 21:35












  • $begingroup$
    Thanks and I edited
    $endgroup$
    – DD90
    Jan 2 at 21:37


















  • $begingroup$
    It is true if you assume that $lim_{x to infty} f'(x)$ exists, see e.g. math.stackexchange.com/questions/42277/…. Also related: math.stackexchange.com/q/313676/42969.
    $endgroup$
    – Martin R
    Jan 2 at 21:35












  • $begingroup$
    Thanks and I edited
    $endgroup$
    – DD90
    Jan 2 at 21:37
















$begingroup$
It is true if you assume that $lim_{x to infty} f'(x)$ exists, see e.g. math.stackexchange.com/questions/42277/…. Also related: math.stackexchange.com/q/313676/42969.
$endgroup$
– Martin R
Jan 2 at 21:35






$begingroup$
It is true if you assume that $lim_{x to infty} f'(x)$ exists, see e.g. math.stackexchange.com/questions/42277/…. Also related: math.stackexchange.com/q/313676/42969.
$endgroup$
– Martin R
Jan 2 at 21:35














$begingroup$
Thanks and I edited
$endgroup$
– DD90
Jan 2 at 21:37




$begingroup$
Thanks and I edited
$endgroup$
– DD90
Jan 2 at 21:37










2 Answers
2






active

oldest

votes


















4












$begingroup$

As stated the result is wrong. Take $f(x)=sin x^2$.






share|cite|improve this answer









$endgroup$













  • $begingroup$
    Sorry I edited.
    $endgroup$
    – DD90
    Jan 2 at 21:52



















2












$begingroup$

Suppose $lim_{x to infty} f'(x) >a >0$. Then $f(n)-f(n-1) > a $ for $n$ suffciently large, say for $n geq n_0$ [This is by MVT]. Then $f(n) geq f(n_0)+ a (n-n_0)$ for $n geq n_0$ which contradicts the hypothesis that $frac {f(x)} xto 0$. For the case $lim_{x to infty} f'(x) <0$ simply replace $f$ by $-f$.






share|cite|improve this answer









$endgroup$




















    2 Answers
    2






    active

    oldest

    votes








    2 Answers
    2






    active

    oldest

    votes









    active

    oldest

    votes






    active

    oldest

    votes









    4












    $begingroup$

    As stated the result is wrong. Take $f(x)=sin x^2$.






    share|cite|improve this answer









    $endgroup$













    • $begingroup$
      Sorry I edited.
      $endgroup$
      – DD90
      Jan 2 at 21:52
















    4












    $begingroup$

    As stated the result is wrong. Take $f(x)=sin x^2$.






    share|cite|improve this answer









    $endgroup$













    • $begingroup$
      Sorry I edited.
      $endgroup$
      – DD90
      Jan 2 at 21:52














    4












    4








    4





    $begingroup$

    As stated the result is wrong. Take $f(x)=sin x^2$.






    share|cite|improve this answer









    $endgroup$



    As stated the result is wrong. Take $f(x)=sin x^2$.







    share|cite|improve this answer












    share|cite|improve this answer



    share|cite|improve this answer










    answered Jan 2 at 21:34









    mathcounterexamples.netmathcounterexamples.net

    25.6k21953




    25.6k21953












    • $begingroup$
      Sorry I edited.
      $endgroup$
      – DD90
      Jan 2 at 21:52


















    • $begingroup$
      Sorry I edited.
      $endgroup$
      – DD90
      Jan 2 at 21:52
















    $begingroup$
    Sorry I edited.
    $endgroup$
    – DD90
    Jan 2 at 21:52




    $begingroup$
    Sorry I edited.
    $endgroup$
    – DD90
    Jan 2 at 21:52











    2












    $begingroup$

    Suppose $lim_{x to infty} f'(x) >a >0$. Then $f(n)-f(n-1) > a $ for $n$ suffciently large, say for $n geq n_0$ [This is by MVT]. Then $f(n) geq f(n_0)+ a (n-n_0)$ for $n geq n_0$ which contradicts the hypothesis that $frac {f(x)} xto 0$. For the case $lim_{x to infty} f'(x) <0$ simply replace $f$ by $-f$.






    share|cite|improve this answer









    $endgroup$


















      2












      $begingroup$

      Suppose $lim_{x to infty} f'(x) >a >0$. Then $f(n)-f(n-1) > a $ for $n$ suffciently large, say for $n geq n_0$ [This is by MVT]. Then $f(n) geq f(n_0)+ a (n-n_0)$ for $n geq n_0$ which contradicts the hypothesis that $frac {f(x)} xto 0$. For the case $lim_{x to infty} f'(x) <0$ simply replace $f$ by $-f$.






      share|cite|improve this answer









      $endgroup$
















        2












        2








        2





        $begingroup$

        Suppose $lim_{x to infty} f'(x) >a >0$. Then $f(n)-f(n-1) > a $ for $n$ suffciently large, say for $n geq n_0$ [This is by MVT]. Then $f(n) geq f(n_0)+ a (n-n_0)$ for $n geq n_0$ which contradicts the hypothesis that $frac {f(x)} xto 0$. For the case $lim_{x to infty} f'(x) <0$ simply replace $f$ by $-f$.






        share|cite|improve this answer









        $endgroup$



        Suppose $lim_{x to infty} f'(x) >a >0$. Then $f(n)-f(n-1) > a $ for $n$ suffciently large, say for $n geq n_0$ [This is by MVT]. Then $f(n) geq f(n_0)+ a (n-n_0)$ for $n geq n_0$ which contradicts the hypothesis that $frac {f(x)} xto 0$. For the case $lim_{x to infty} f'(x) <0$ simply replace $f$ by $-f$.







        share|cite|improve this answer












        share|cite|improve this answer



        share|cite|improve this answer










        answered Jan 2 at 23:51









        Kavi Rama MurthyKavi Rama Murthy

        53.5k32055




        53.5k32055















            Popular posts from this blog

            android studio warns about leanback feature tag usage required on manifest while using Unity exported app?

            SQL update select statement

            WPF add header to Image with URL pettitions [duplicate]